IMO 2007 - P1

Problemas que aparecen en el Archivo de Enunciados.
Avatar de Usuario
Gianni De Rico

FOFO 7 años - Mención Especial-FOFO 7 años OFO - Medalla de Oro-OFO 2019 FOFO 9 años - Jurado-FOFO 9 años COFFEE - Jurado-COFFEE Matías Saucedo OFO - Jurado-OFO 2020
FOFO Pascua 2020 - Jurado-FOFO Pascua 2020 COFFEE - Jurado-COFFEE Carolina González COFFEE - Jurado-COFFEE Ariel Zylber COFFEE - Jurado-COFFEE Iván Sadofschi FOFO 10 años - Jurado-FOFO 10 años
OFO - Jurado-OFO 2021 FOFO 11 años - Jurado-FOFO 11 años OFO - Jurado-OFO 2022 FOFO Pascua 2022 - Jurado-FOFO Pascua 2022 FOFO 12 años - Jurado-FOFO 12 años
OFO - Jurado-OFO 2023 FOFO 13 años - Jurado-FOFO 13 años OFO - Jurado-OFO 2024
Mensajes: 2212
Registrado: Vie 16 Sep, 2016 6:58 pm
Medallas: 18
Nivel: Exolímpico
Ubicación: Rosario
Contactar:

IMO 2007 - P1

Mensaje sin leer por Gianni De Rico »

Sean $a_1,a_2,\ldots ,a_n$ números reales. Para cada $i$ ($1\leq i\leq n$) se define$$d_i=\max \{a_j:1\leq j\leq i\}-\min \{a_j:i\leq j\leq n\}$$y sea$$d=\max \{d_i:1\leq i\leq n\}.$$(a) Demostrar que para cualesquiera números reales $x_1\leq x_2\leq \cdots \leq x_n$,$$\max \{|x_i-a_i|:1\leq i\leq n\}\geq \frac{d}{2}.$$(b) Demostrar que existen números reales $x_1\leq x_2\leq \cdots \leq x_n$ para los cuales se cumple la igualdad.
♪♫ do re mi función lineal ♪♫
BrunZo

OFO - Medalla de Bronce-OFO 2018 FOFO 8 años - Mención Especial-FOFO 8 años OFO - Medalla de Plata-OFO 2019 FOFO Pascua 2019 - Medalla-FOFO Pascua 2019 FOFO 9 años - Medalla Especial-FOFO 9 años
OFO - Medalla de Oro-OFO 2020 FOFO Pascua 2020 - Copa-FOFO Pascua 2020 FOFO 10 años - Copa-FOFO 10 años OFO - Medalla de Oro-OFO 2021 FOFO 11 años - Medalla-FOFO 11 años
OFO - Medalla de Oro-OFO 2022 FOFO Pascua 2022 - Medalla-FOFO Pascua 2022 FOFO 12 años - Medalla-FOFO 12 años OFO - Jurado-OFO 2023 FOFO 13 años - Jurado-FOFO 13 años
OFO - Jurado-OFO 2024
Mensajes: 414
Registrado: Mar 21 Nov, 2017 8:12 pm
Medallas: 16
Nivel: 3

Re: IMO 2007 - P1

Mensaje sin leer por BrunZo »

Solución:
Spoiler: mostrar
Pensando un tiempo el enunciado, podemos notar que el valor de $d$ es, en realidad, la mayor diferencia $a_i-a_j$ con $i\leq j$.
Además, el problema nos pide demostrar que existe algún $a_i$ tal que $|a_i-x_i|$ es mayor que $d$.

Para resolver la parte a, tomamos la dicha mayor diferencia $d=a_i-a_j$ y consideramos los correspondientes $x_i\leq x_j$.
Vamos a asumir que $d$ es positivo, ya que si no, como el lado izquierdo de la desigualdad es no negativo, el problema se vuelve trivial.
Ahora, supongamos que $|a_i-x_i|< \frac{d}{2}$, $|a_j-x_j|< \frac{d}{2}$. Entonces, sabemos que
$x_i\in (a_i-\frac{d}{2};a_i+\frac{d}{2})$ y $x_j\in (a_j-\frac{d}{2};a_j+\frac{d}{2})$
Pero
$$d\geq 0\Longleftrightarrow a_j<a_i\Longleftrightarrow \left(a_j-\frac{d}{2}\right)<x_j<\left(a_j+\frac{d}{2}\right)=\left(a_i-\frac{d}{2}\right)<x_i<\left(a_i+\frac{d}{2}\right)$$
luego $x_j<x_i$, lo cual es absurdo. De esto se deduce que uno de $|a_i-x_i|$, $|a_j-x_j|$ es mayor o igual a $\frac{d}{2}$, con lo que se resuelve la parte a.

Para resolver la parte b, damos un ejemplo de sucesión:
Para todos los $x_k$ con $i\leq k\leq j$ (estamos usando $i$ e $j$ de la parte a), $x_k=\frac{a_i+a_j}{2}$. Como todos los $a_k$ de este tipo están en el intervalo $(a_i;a_j)$ (ya que $a_i-a_j$ es máximo), es claro que $|a_k-x_k|\leq\frac{d}{2}$.
Ahora, consideramos los grupos de elementos anteriores a $a_i$ y los posteriores a $a_j$.
Usando el argumento de que $a_i-a_j$ es máximo, sabemos que todos los números anteriores a $a_i$ son menores que $a_i$, y que los posteriores a $a_j$ son mayores que $a_j$.
Entonces, para cada $a_m$ con $m>j$, vamos a tomar
$x_m=x_{m-1}$ si $|a_m-x_{m-1}|\leq\frac{d}{2}$ ó $x_m=a_m-\frac{d}{2}$ si no.
Veamos que esto cumple, puesto que cuando reasignamos $x_m$ (o sea, cuando poner $x_m=x_{m+1}$ causa que $|a_m-x_m>\frac{d}{2}$), entonces, hasta que volvamos a reasignarlo, todos los valores de $a_m$ van a estar en el intervalo $(a_m-d;a_m)$, por lo que el valor de $x_m=a_m-\frac{d}{2}$ funciona.
Lo mismo para los $m<i$.
Y con el ejemplo armado, finalizamos el problema.
Responder